ATI Testbank Questions- OB Exam #2 part I

अब Quizwiz के साथ अपने होमवर्क और परीक्षाओं को एस करें!

Which of the following is the priority intervention for the client in a left side-lying position whose monitor strip shows a deceleration that extends beyond the end of the contraction? a. Administer O2 at 8 to 10 L/min. b. Decrease the IV rate to 100 mL/hr. c. Reposition the ultrasound transducer. d. Perform a vaginal exam to assess for cord prolapse.

a. Administer O2 at 8 to 10 L/min. A deceleration that returns to baseline after the end of the contraction is a late deceleration caused by placental perfusion problems. Administering oxygen will increase the clients blood oxygen saturation, making more oxygen available to the fetus. Decreasing the IV rate, repositioning the ultrasound transducer, and performing a vaginal exam to assess for cord prolapse are not effective interventions to improve fetal oxygenation.

An assessment finding that would indicate to the nurse that cervical dilation and/or effacement has occurred is: a. onset of irregular contractions. b. cephalic presentation at 0 station. c. bloody mucus drainage from vagina. d. fetal heart tones (FHTs) present in the lower right quadrant.

c. bloody mucus drainage from vagina. Cervical dilation and/or effacement results in loss of the mucous plug as well as rupture of small capillaries in the cervix; irregular contractions, cephalic presentation, and FHTs in the lower right quadrant do not indicate the onset of cervical ripening.

The nurse has given the newborn an Apgar score of 5. She should then: a. begin ventilation and compressions. b. do nothing except place the infant under a radiant warmer. c. observe the infant and recheck the score after 10 minutes. d. gently stimulate by rubbing the infants back while administering O2.

d. gently stimulate by rubbing the infants back while administering O2. An infant who receives a score of 4 to 6 requires only additional oxygen and gentle stimulation. An infant who receive a score of 3 or less requires ventilation and compressions. An infant who scores less than 7 requires more intervention than placement under a radiant warmer. Observing and rechecking the infant will not improve newborns transition to extrauterine life.

A client in labor presents with a breech presentation. The nurse understands that a breech presentation is associated with: a. more rapid labor. b. a high risk of infection. c. maternal perineal trauma. d. umbilical cord compression.

d. umbilical cord compression. The umbilical cord can compress between the fetal body and maternal pelvis when the body has been born but the head remains within the pelvis. Breech presentation is not associated with a more rapid labor. There is no higher risk of infection with a breech birth. There is no higher risk for perineal trauma with a breech birth.

Pregnant clients can usually tolerate the normal blood loss associated with childbirth because they have: a. a higher hematocrit. b. increased leukocytes. c. increased blood volume. d. a lower fibrinogen level.

c. increased blood volume. Women have a significant increase in blood volume during pregnancy. After birth, the additional circulating volume is no longer necessary. The hematocrit decreases with pregnancy because of the high fluid volume. Leukocyte levels increase during labor, but that is not the reason for the toleration of blood loss. Fibrinogen levels increase with pregnancy.

Which client has the priority need for fetal monitoring? a. Primigravida at 38 weeks with spontaneous ROM b. Multigravida at 40 weeks with history of 10-hour labors c. Multigravida admitted for repeat elective cesarean section d. Primigravida at 39 weeks with meconium-stained amniotic fluid

d. Primigravida at 39 weeks with meconium-stained amniotic fluid Meconium-stained amniotic fluid indicates a potential risk factor during labor. Primigravida at 38 weeks with spontaneous ROM, multigravida with a history of 10-hour labors, and multigravida admitted for repeat elective cesarean section do not have potential maternal or fetal risk factors.

The nurse is preparing to initiate intravenous (IV) access on a patient in the active phase of labor. Which size IV cannula is best for this patient? a. 18-gauge b. 20-gauge c. 22-gauge d. 24-gauge

a. 18-gauge The larger the number, the smaller the diameter of the cannula. The nurse should select the largest bore cannula possible. IV access is initiated for hydration prior to epidural placement and for use in an emergency. Both require the rapid administration of fluid, which is most easily accomplished with a large bore cannula.

The assessment finding which indicates that the client is in the active phase of the first stage of labor is: a. 80% effacement. b. dilation of 5 cm. c. presence of bloody show. d. regular contraction every 3 to 4 minutes.

b. dilation of 5 cm. The active phase of labor is defined by cervical dilation between 4 to 7 cm. Effacement, bloody show, and regular contractions are not parameters whereby the phases of labor are defined.

The nurse is monitoring a client in labor and notes this fetal heart rate pattern on the electronic fetal monitoring strip (see figure). Which is the most appropriate nursing action? a. Administer oxygen with a face mask at 8 to 10 L/min. b. Reposition the fetal monitor ultrasound transducer. c. Assist the client to the bathroom to empty her bladder . d. Continue to monitor the client and fetal heart rate patterns.

a. Administer oxygen with a face mask at 8 to 10 L/min. Late decelerations are similar to early decelerations in the degree of FHR slowing and lowest rate (30 to 40 bpm) but are shifted to the right in relation to the contraction. They often begin after the peak of the contraction. They reflect possible impaired placental exchange (uteroplacental insufficiency). Administration of 100% oxygen through a snug face mask makes more oxygen available for transfer to the fetus. A commonly suggested rate is 8 to 10 L/min. The pattern is nonreassuring so repositioning the fetal ultrasound transducer, assisting the client to the bathroom, or continuing to monitor the pattern will not correct the problem.

The nurse observes the following data on an electronic fetal monitor attached to a client in the active phase of the first stage of labor: fetal heart rate baseline, 125 to 140 bpm, three accelerations over the course of 20 minutes, moderate variability. What is the priority action based on these findings? a. Document the findings. b. Contact the health care provider. c. Increase the rate of the existing IV to 200 mL/hr as per the standing prescription. d. Place oxygen via a rebreather mask at 10 L/min as per the standing prescription.

a. Document the findings. The findings are all within normal limits for the laboring client. Accelerations are usually a reassuring sign. Normal fetal heart rate is 110 to 160 bpm and of moderate variability; amplitude range of 6 to 25 bpm is desirable. No intervention is required because the pattern suggests that the fetus has adequate reserves to tolerate intrapartum stressors.

The nurse is preparing supplies for an amnioinfusion on a client with intact membranes. Which supplies should the nurse gather? (Select all that apply.) a. Extra underpads b. Solution of 3% normal saline c. Amniotic hook to perform an amniotomy d. Solid intrauterine pressure catheter with a pressure transducer on its tip

a. Extra underpads c. Amniotic hook to perform an amniotomy Amnioinfusion is performed with lactated Ringers solution or normal saline, not 3%. Normal saline is infused into the uterus through an intrauterine pressure catheter (IUPC). The underpads must be changed regularly because fluid leaks out constantly. The membranes need to be ruptured before an amnioinfusion can be initiated so an amniotic hook will be needed. The IUPC must have a double lumen to run the infusion through.

The nurse is reviewing an electronic fetal monitor tracing from a patient in active labor and notes the fetal heart rate gradually drops to 20 beats per minute (bpm) below the baseline and returns to the baseline well after the completion of the patients contractions. How will the nurse document these findings? a. Late decelerations b. Early decelerations c. Variable decelerations d. Proximal decelerations

a. Late decelerations Late decelerations are similar to early decelerations in the degree of FHR slowing and lowest rate (30 to 40 bpm) but are shifted to the right in relation to the contraction. They often begin after the peak of the contraction. The FHR returns to baseline after the contraction ends. The early decelerations mirror the contraction, beginning near its onset and returning to the baseline by the end of the contraction, with the low point (nadir) of the deceleration occurring near the contractions peak. The rate at the lowest point of the deceleration is usually no lower than 30 to 40 bpm from the baseline. Conditions that reduce flow through the umbilical cord may result in variable decelerations. These decelerations do not have the uniform appearance of early and late decelerations. Their shape, duration, and degree of fall below baseline rate vary. They fall and rise abruptly (within 30 seconds) with the onset and relief of cord compression, unlike the gradual fall and rise of early and late decelerations. Proximal decelerations is not a recognized term.

Which client is a candidate for internal monitoring with an intrauterine pressure catheter? a. Obese client whose contractions are 3 to 6 minutes apart, lasting 20 to 50 seconds b. Gravida 1, para 0, whose contractions are 2 to 3 minutes apart, lasting 60 seconds c. Multigravida whose contractions are 2 minutes apart, lasting 60 to 70 seconds d. Gravida 2, para 1, in latent phase whose contractions are irregular and mild

a. Obese client whose contractions are 3 to 6 minutes apart, lasting 20 to 50 seconds A thick layer of abdominal fat absorbs energy from uterine contractions, reducing their apparent intensity on the monitor strip. Contraction patterns of 2 to 3 minutes lasting 60 seconds and every 2 minutes lasting 60 to 70 seconds indicate accurate measurement of uterine activity. Irregular and mild contractions are common in the latent phase.

38. Which medications could potentially cause hyperstimulation of the uterus during labor? (Select all that apply.) a. Oxytocin (Pitocin) b. Misoprostol (Cytotec) c. Dinoprostone (Cervidil) d. Methylergonovine maleate (Methergine)

a. Oxytocin (Pitocin) b. Misoprostol (Cytotec) c. Dinoprostone (Cervidil) d. Methylergonovine maleate (Methergine) Oxytocin, misoprostol, and dinoprostone fall under the general category of uterine stimulants. Cytotec and Cervidil are prostaglandins. Methergine is an ergot alkaloid.

A patient at 41 weeks gestation is undergoing an induction of labor with an IV administration of oxytocin (Pitocin). The fetal heart rate starts to demonstrate a recurrent pattern of late decelerations with moderate variability. What is the nurses priority action? a. Stop the infusion of Pitocin. b. Reposition the patient from her right to her left side. c. Perform a vaginal exam to assess for a prolapsed cord. d. Prepare the patient for an emergency cesarean section.

a. Stop the infusion of Pitocin. There are multiple reasons for late decelerations. Address the probable cause first, such as uterine hyperstimulation with Pitocin, to alleviate the outcome of late decelerations. Repositioning can increase oxygenation to the fetus but does not address the cause of the problem. Variable decelerations are more often seen with a prolapsed cord. In the presence of moderate variability, the fetus continues to have adequate oxygen reserves. The presence of two or more nonreassuring fetal heart rate patterns increases the level of concern.

When evaluating the clients progress, the nurse knows that four of the five fetal factors that interact to regulate the heart rate are (select all that apply): a. baroreceptors. b. adrenal glands. c. chemoreceptors. d. uterine activity. e. autonomic nervous system.

a. baroreceptors. b. adrenal glands. c. chemoreceptors. e. autonomic nervous system. The sympathetic and parasympathetic branches of the autonomic nervous system are balanced forces that regulate FHR. Sympathetic stimulation increases the heart rate, whereas parasympathetic responses, through stimulation of the vagus nerve, reduce the FHR and maintain variability. The baroreceptors stimulate the vagus nerve to slow the FHR and decrease the blood pressure. These are located in the carotid arch and major arteries. The chemoreceptors are cells that respond to changes in oxygen, carbon dioxide, and pH. They are found in the medulla oblongata and aortic and carotid bodies. The adrenal medulla secretes epinephrine and norepinephrine in response to stress, causing accelerations in FHR. Hypertonic uterine activity can reduce the time available for the exchange of oxygen and waste products; however, this is a maternal factor. The fifth fetal factor is the central nervous system. The fetal cerebral cortex causes the heart rate to increase during fetal movement and decrease when the fetus sleeps.

Decelerations that mirror the contractions are present with each contraction on the monitor strip of a multipara who received epidural anesthesia 20 minutes ago. The nurse should: a. maintain the normal assessment routine. b. administer O2 at 8 to 10 L/min by face mask. c. increase the IV flow rate from 125 to 150 mL/hr. d. assess the maternal blood pressure for a systolic pressure below 100 mm Hg.

a. maintain the normal assessment routine. Decelerations that mirror the contraction are early decelerations caused by fetal head compression. Early decelerations are not associated with fetal compromise and require no intervention. Administering O2, increasing the IV flow rate, and assessing for hypotension are not necessary in early decelerations.

The physician has ordered an amnioinfusion for the laboring client. What data supports the use of this therapeutic procedure? a. Presenting part not engaged b. +4 meconium-stained amniotic fluid on artificial rupture of membranes (AROM) c. Breech position of fetus d. Twin gestation

b. +4 meconium-stained amniotic fluid on artificial rupture of membranes (AROM) Amnioinfusion is a procedure used during labor when cord compression or the detection of gross meconium staining is found in the amniotic fluid. A saline solution is used as an irrigation method through the IUPC (intrauterine pressure catheter).

What is the most likely cause for this fetal heart rate pattern? a. Administration of an epidural for pain relief during labor b. Cord compression c. Breech position of fetus d. Administration of meperidine (Demerol) for pain relief during labor

b. Cord compression Variable deceleration patterns are seen in response to head compression or cord compression. A breech presentation would not be likely to cause this fetal heart rate pattern. Similarly, administration of medication and/or an epidural would not cause this fetal heart rate pattern.

The nurse recognizes that fetal scalp stimulation may be prescribed to evaluate the response of the fetus to tactile stimulation. Which conditions contraindicate the use of fetal scalp stimulation? (Select all that apply.) a. Post-term fetus b. Maternal fever c. Placenta previa d. Induction of labor e. Prolonged rupture of membranes

b. Maternal fever c. Placenta previa e. Prolonged rupture of membranes Fetal scalp stimulation is not done when there is maternal fever (possibility of introducing microorganisms into the uterus), placenta previa (placenta overlies the cervix, and hemorrhage is likely), or prolonged rupture of membranes (risk of infection). Fetal scalp stimulation may be used to evaluate a post-term fetus response to stimulation. It is also used to evaluate a fetus when labor is being induced.

Client is at 38 weeks gestation, gravida 1, para 0, vaginal exam4 cm, 100% effaced, +1 station vertex. What is the most likely intervention for this fetal heart rate pattern? a. Continue oxytocin (Pitocin) infusion. b. Contact the anesthesia department for epidural administration. c. Change maternal position. d. Administer Narcan to client and prepare for immediate vaginal delivery.

c. Change maternal position. Late decelerations indicate fetal compromise (uteroplacental insufficiency) and are considered to be a significant event requiring immediate assessment and intervention. Of all the options listed, changing maternal position may increase placental perfusion. In the presence of late decelerations, Pitocin infusion should be stopped. Contacting anesthesia for epidural administration will not solve the existing problem of late decelerations. There are no data to support the administration of Narcan and because client is still in early labor, birth is not imminent.

The nurse admits a laboring patient at term. On review of the prenatal record, the patients pregnancy has been unremarkable and she is considered low risk. In planning the patients care, at what interval will the nurse intermittently auscultate (IA) the fetal heart rate during the first stage of labor? a. Every 10 minutes b. Every 15 minutes c. Every 30 minutes d. Every 60 minutes

c. Every 30 minutes Evaluate the fetal monitoring strip systematically for the elements noted. The following are recommended assessment and documentation intervals for IA and EFM (although facility policies may be different): low-risk women, every 30 minutes during the active phase and every 15 minutes during the second stage.

Which of the following therapeutic applications provides the most accurate information related to uterine contraction strength? a. External fetal monitoring (EFM) b. Internal fetal monitoring c. Intrauterine pressure catheter (IUPC) d. Maternal comments based on perception

c. Intrauterine pressure catheter (IUPC) IUPC is a clinical tool that provides an accurate assessment of uterine contraction strength. EFM provides evidence of contraction pattern and fetal heart rate but only estimates uterine contraction strength. Internal fetal monitoring provides direct evidence of fetal heart rate and contraction pattern. It only estimates uterine contraction strength. Maternal comments related to pain may not be related to uterine contraction strength and thus are influenced by the clients own pain perception.

Which of the following is the priority intervention for a supine client whose monitor strip shows decelerations that begin after the peak of the contraction and return to the baseline after the contraction ends? a. Increase IV infusion. b. Elevate lower extremities. c. Reposition to left side-lying position. d. Administer oxygen per face mask at 4 to 6 L/min.

c. Reposition to left side-lying position. Decelerations that begin at the peak of the contractions and recover after the contractions end are caused by uteroplacental insufficiency. When the client is in the supine position, the weight of the uterus partially occludes the vena cava and descending aorta, resulting in hypotension and decreased placental perfusion. Increasing the IV infusion, elevating the lower extremities, and administering O2 will not be effective as long as the client is in a supine position.

To clarify the fetal condition when baseline variability is absent, the nurse should first: a. monitor fetal oxygen saturation using fetal pulse oximetry. b. notify the physician so that a fetal scalp blood sample can be obtained. c. apply pressure to the fetal scalp with a glove finger using a circular motion. d. increase the rate of non-additive IV fluid to expand the mothers blood volume.

c. apply pressure to the fetal scalp with a glove finger using a circular motion. Fetal scalp stimulation helps identify whether the fetus responds to gentle massage. An acceleration in response to the massage suggests that the fetus is in normal oxygen and acid-base balance. Monitoring fetal oxygen saturation using fetal pulse oximetry is no longer available in the United States. Obtaining a fetal scalp blood sample is invasive and the results are not immediately available. Increasing the rate of non-additive IV fluid would not clarify the fetal condition.

The nurse is monitoring a client in labor and notes this fetal heart rate pattern on the electronic fetal monitoring strip (see figure). Which is the most appropriate nursing action? a. Decrease the rate of the IV fluids. b. Document the fetal heart rate pattern. c. Explain to the client that the pattern is reassuring. d. Perform a vaginal exam to detect a prolapsed cord.

d. Perform a vaginal exam to detect a prolapsed cord. Variable decelerations do not have the uniform appearance of early and late decelerations. Their shape, duration, and degree of fall below baseline rate vary. They fall and rise abruptly (within 30 seconds) with the onset and relief of cord compression, unlike the gradual fall and rise of early and late decelerations. A vaginal examination may identify a prolapsed cord, which may cause variable decelerations, bradycardia, or both as it is compressed. A vaginal examination also evaluates the woman's labor status, which helps the birth attendant decide if labor should continue. This is a nonreassuring pattern so the IV rate should be increased and an intervention needs to be done, not just documentation.

Observation of a fetal heart rate pattern indicates an increase in heart rate from the prior baseline rate of 152 bpm. Which physiologic mechanisms would account for this situation? a. Inhibition of epinephrine b. Inhibition of norepinephrine c. Stimulation of the vagus nerve d. Sympathetic stimulation

d. Sympathetic stimulation Sympathetic nerve innervation would result in an increase in fetal heart rate. The release of epinephrine as a result of sympathetic innervation would lead to an increase in fetal heart rate. The release of norepinephrine as a result of sympathetic innervation would lead to an increase in fetal heart rate. Stimulation of the vagus nerve would indicate parasympathetic innervation and result in a decreased heart rate.

When a pattern of variable decelerations occur, the nurse should: a. administer O2 at 8 to 10 L/min. b. place a wedge under the right hip. c. increase the IV fluids to 150 mL/hr. d. position client in a knee-chest position.

d. position client in a knee-chest position. Variable decelerations are caused by conditions that reduce flow through the umbilical cord. The client should be repositioned when the FHR pattern is associated with cord compression. The knee-chest position uses gravity to shift the fetus out of the pelvis to relieve cord compression. Administering oxygen will not be effective until cord compression is relieved. Increasing the IV fluids and placing a wedge under the right hip are not effective interventions for cord compression.

The nurse in the birth room receives an order to give a newborn 0.3 mg of naloxone (Narcan) intramuscularly. The medication vial reads naloxone (Narcan), 0.4 mg/mL. The nurse should prepare how many milliliters to administer the correct dose? Fill in the blank and record your answer using two decimal places.

0.75 mL Desired/available volume = milliliters per dose (0.3 mg/0.4 mg) 1 mL = 0.75 mL/dose

Which maternal factor may inhibit fetal descent? a. A full bladder b. Decreased peristalsis c. Rupture of membranes d. Reduction in internal uterine size

a. A full bladder A full bladder may inhibit fetal descent because it occupies space in the pelvis needed by the fetal presenting part. Peristalsis does not influence fetal descent. Rupture of membranes will assist in the fetal descent. Contractions will reduce the internal uterine size to assist fetal descent.

Which should the nurse expect to assess in the third stage of labor that indicates the placenta has separated from the uterine wall? (Select all that apply.) a. A gush of blood appears. b. The uterus rises upward in the abdomen. c. The fundus descends below the umbilicus. d. The cord descends further from the vagina. e. The uterus becomes boggy and soft, with an elongated shape.

a. A gush of blood appears. b. The uterus rises upward in the abdomen. d. The cord descends further from the vagina. Four signs suggest placenta separation. The uterus has a spherical shape. The uterus rises upward in the abdomen as the placenta descends into the vagina and pushes the fundus upward. The cord descends further from the vagina. A gush of blood appears as blood trapped behind the placenta is released. The fundus rises upward above the umbilicus. A boggy uterus with an elongated shape would not be expected.

A 28-year-old gravida 1, para 0 client who is at term calls the labor and birth unit stating that she thinks she is in labor. She states that she does have some vaginal discharge and feels wet but it is not bloody in nature. She relates a contraction pattern that is irregular, ranging from 5 to 7 minutes and lasting 30 seconds. What questions would be used during the process of phone triage by the nurse? (Select all that apply.) a. Ask her if her if she thinks that her membranes have ruptured. b. Ask her if she has any evidence of bloody show. c. Have her keep monitoring her contraction pattern and call you back if they become more regular. d. Ask her when her she has her next scheduled visit with her health care provider. e. Tell her to come into the hospital for evaluation.

a. Ask her if her if she thinks that her membranes have ruptured. e. Tell her to come into the hospital for evaluation. The cornerstone of obstetric triage is reassurance of maternal-fetal well-being. Thus, in view of the assessment data that the client provided, the nurse should ascertain membrane status and ask the client to come in for evaluation. The client has already indicated that the vaginal discharge was not bloody in nature. Having the client continue to monitor at home would not provide assurance of maternal-fetal well-being. Asking the client about the next scheduled physician visit does not address current health concerns of impending labor.

Which clinical findings would be considered to be normal for a preterm fetus during the labor period? a. Baseline tachycardia b. Baseline bradycardia c. Fetal anemia d. Acidosis

a. Baseline tachycardia Because the nervous system is immature, it is expected that the preterm fetus will have a baseline tachycardia because of stimulation of the sympathetic nervous system. Baseline bradycardia, fetal anemia, and acidosis would indicate abnormal findings and fetal compromise.

The nurse is instructing a nursing student on the application of fetal monitoring devices. Which method of assessing the fetal heart rate requires the use of a gel? a. Doppler b. Fetoscope c. Scalp electrode d. Tocodynamometer

a. Doppler Doppler is the only listed method involving ultrasonic transmission of fetal heart rates; it requires the use of a gel. The fetoscope does not require gel because ultrasonic transmission is not used. The scalp electrode is attached to the fetal scalp; gel is not necessary. The tocodynamometer does not require gel. This device monitors uterine contractions.

Which assessment finding could indicate hemorrhage in the postpartum patient? a. Elevated pulse rate b. Elevated blood pressure c. Firm fundus at the midline d. Saturation of two perineal pads in 4 hours

a. Elevated pulse rate An increasing pulse rate is an early sign of excessive blood loss. If the blood volume were diminishing, the blood pressure would decrease. A firm fundus indicates that the uterus is contracting and compressing the open blood vessels at the placental site. Saturation of one pad within the first hour is the maximum normal amount of lochial flow. Two pads within 4 hours is within normal limits.

The client in labor experiences a spontaneous rupture of membranes. What information related to this event must the nurse include in the clients record? a. Fetal heart rate b. Pain level c. Test results ensuring that the fluid is not urine d. The clients understanding of the event

a. Fetal heart rate Charting related to membrane rupture includes the time, FHR, and character and amount of the fluid. Pain is not associated with this event. When it is obvious that the fluid is amniotic fluid, which is anticipated during labor, it is not necessary to verify this by testing. The clients understanding of the event would only need to be documented if it presents a problem.

Which is an essential part of nursing care for a laboring client? a. Helping the woman manage the pain b. Eliminating the pain associated with labor c. Feeling comfortable with the predictable nature of intrapartal care d. Sharing personal experiences regarding labor and birth to decrease her anxiety

a. Helping the woman manage the pain Helping a client manage the pain is an essential part of nursing care because pain is an expected part of normal labor and cannot be fully relieved. Labor pain cannot be fully relieved. The labor nurse should always be assessing for unpredictable occurrences. Decreasing anxiety is important, but managing pain is a top priority.

A client asks the nurse how she can tell if labor is real? What should the nurse give as an explanation? (Select all that apply.) a. In true labor, the cervix begins to dilate. b. In true labor, the contractions are felt in the abdomen and groin. c. In true labor, contractions often resemble menstrual cramps during early labor. d. In true labor, contractions are inconsistent in frequency, duration, and intensity in the early stages. e. In true labor your contractions tend to increase in frequency, duration, and intensity with walking.

a. In true labor, the cervix begins to dilate. c. In true labor, contractions often resemble menstrual cramps during early labor. e. In true labor your contractions tend to increase in frequency, duration, and intensity with walking. In true labor, the cervix begins to dilate, contractions often resemble menstrual cramps in the early stage, and labor contractions increase in frequency, duration, and intensity with walking. False labor contractions are felt in the abdomen and groin and the contractions are inconsistent in frequency, duration, and intensity.

The nurse auscultates the fetal heart rate and determines a rate of 152 bpm. Which nursing intervention is appropriate? a. Inform the mother that the rate is normal. b. Reassess the fetal heart rate in 5 minutes because the rate is too high. c. Report the fetal heart rate to the physician or nurse-midwife immediately. d. Tell the mother that she is going to have a boy because the heart rate is fast.

a. Inform the mother that the rate is normal. The FHR is within the normal range, so no other action is indicated at this time. The FHR is within the expected range; reassessment should occur, but not in 5 minutes. The FHR is within the expected range; no further action is necessary at this point. The gender of the baby cannot be determined by the FHR.

A client just delivered a baby by the vaginal route. The client asks the nurse why the baby's head is not round, but oval. Which explanation should the nurse give to the client? a. This results from molding. b. This results from lightening. c. This results from the fetal lie. d. This results from the fetal presentation.

a. This results from molding. The sutures and fontanels allow the bones of the fetal head to move slightly, changing the shape of the fetal head so it can adapt to the size and shape of the pelvis. Lightening is the descent of the fetus toward the pelvic inlet before labor. Lie is the relationship of the long axis of the fetus to the long axis of the mother. Presentation is the fetal part that first enters the pelvic outlet.

A laboring client is 10 cm dilated but does not feel the urge to push. The nurse understands that according to laboring down, the advantages of waiting until an urge to push are which of the following? (Select all that apply.) a. Less maternal fatigue b. Less birth canal injuries c. Decreased pushing time d. Faster descent of the fetus e. An increase in frequency of contractions

a. Less maternal fatigue b. Less birth canal injuries c. Decreased pushing time Delayed pushing has been shown to result in less maternal fatigue and decreased pushing time. Pushing vigorously sooner than the onset of the reflexive urge may contribute to birth canal injury because her vaginal tissues are stretched more forcefully and rapidly than if she pushed spontaneously and in response to her body's signals. A brief slowing of contractions often occurs at the beginning of the second stage.

The nurse is monitoring a client in the active stage of labor. Which conditions associated with fetal compromise should the nurse monitor? (Select all that apply.) a. Maternal hypotension b. Fetal heart rate of 140 to 150 bpm c. Meconium-stained amniotic fluid d. Maternal fever 38 C (100.4 F) or higher e. Complete uterine relaxation of more than 30 seconds between contractions

a. Maternal hypotension c. Meconium-stained amniotic fluid d. Maternal fever38 C (100.4 F) or higher Conditions associated with fetal compromise include maternal hypotension (may divert blood flow away from the placenta to ensure adequate perfusion of the maternal brain and heart), meconium-stained (greenish) amniotic fluid, and maternal fever (38 C [100.4 F] or higher). Fetal heart rate of 110 to 160 bpm for a term fetus is normal. Complete uterine relaxation is a normal finding.

Which interventions are required following an amniotomy procedure? (Select all that apply.) a. Notation related to amount of fluid expelled b. Color and consistency of fluid c. Fetal heart rate d. Maternal blood pressure e. Maternal heart rate

a. Notation related to amount of fluid expelled b. Color and consistency of fluid c. Fetal heart rate Following amniotomy (AROM), observation and documentation of the amount of fluid, color and consistency, and fetal heart rate should be done. Maternal assessments related to blood pressure and heart rate are not required.

The nurse is planning care for a client during the fourth stage of labor. Which interventions should the nurse plan to implement? (Select all that apply.) a. Offer the client a warm blanket. b. Place an ice pack on the perineum. c. Massage the uterus if it is boggy. d. Delay breastfeeding until the client is rested. e. Explain to the client that the lochia will be light pink in color.

a. Offer the client a warm blanket. b. Place an ice pack on the perineum. c. Massage the uterus if it is boggy. The fourth stage of labor lasts from the birth of the placenta through the first 1 to 4 hours after birth. Many women are chilled after birth. A warm blanket, hot drink, or soup may help relieve the chill and make the woman more comfortable. Localized discomfort from birth trauma such as lacerations, episiotomy, edema, or hematoma is evident as the effects of local and regional anesthetics diminish. Ice packs on the perineum limit this edema and hematoma formation. A soft (boggy) uterus and increasing uterine size are associated with postpartum hemorrhage because large blood vessels at the placenta site are not compressed. The uterus should be massaged if it is not firm. The fourth stage is the best time to initiate breastfeeding if maternal and infant problems are absent. The vaginal drainage after childbirth is called lochia. The three stages are lochia rubra, lochia serosa, and lochia alba. Lochia rubra, consisting mostly of blood, is present in the fourth stage of labor. The color of the lochia will be bright red not pink.

After birth of the placenta the patient states, All of a sudden I feel very cold. What is the best nursing action in response to this statement? a. Place a warm blanket over the patient. b. Place the baby on the patients abdomen. c. Tell the patient that chills are expected after birth. d. What do you mean by your words very cold?

a. Place a warm blanket over the patient. Many women are chilled after birth. The cause of this reaction is unknown but probably relates to the sudden decrease in effort, loss of the heat produced by the fetus, decrease in intraabdominal pressure, and fetal blood cells entering the maternal circulation. The chill lasts for about 20 minutes and subsides spontaneously. A warm blanket, hot drink, or soup may help relieve the chill and make the woman more comfortable. Placing the baby on her abdomen may result in transfer of heat and make her feel even colder. Reassurance is appropriate after the blanket is provided. Validation of an expected physical response to the birthing process results in a delay of care and is unnecessary.

The nurse who elects to practice in the area of obstetrics often hears discussion regarding the four Ps. What are the four Ps that interact during childbirth? (Select all that apply.) a. Powers b. Passage c. Position d. Passenger e. Psyche

a. Powers b. Passage d. Passenger e. Psyche Powers: The two powers of labor are uterine contractions and pushing efforts. During the first stage of labor, through full cervical dilation, uterine contractions are the primary force moving the fetus through the maternal pelvis. At some point after full dilation, the woman adds her voluntary pushing efforts to propel the fetus through the pelvis. Passage: The passage for birth of the fetus consists of the maternal pelvis and its soft tissues. The bony pelvis is more important to the successful outcome of labor because bones and joints do not yield as readily to the forces of labor. Passenger: This is the fetus plus the membranes and placenta. Fetal lie, attitude, presentation, and position are all factors that affect the fetus as passenger. Psyche: The psyche is a crucial part of childbirth. Marked anxiety, fear, or fatigue decreases the woman's ability to cope.

Which comfort measure should a nurse use to assist a laboring woman to relax? a. Recommend frequent position changes. b. Palpate her filling bladder every 15 minutes. c. Offer warm wet cloths to use on the clients face and neck. d. Keep the room lights lit so the client and her coach can see everything.

a. Recommend frequent position changes. Frequent maternal position changes reduce the discomfort from constant pressure and promote fetal descent. A full bladder intensifies labor pain. The bladder should be emptied every 2 hours. Women in labor get hot and perspire. Cool cloths are much better. Soft indirect lighting is more soothing than irritating bright lights.

The nurse is caring for a client in the fourth stage of labor. Which assessment findings should the nurse identify as a potential complication? (Select all that apply.) a. Soft boggy uterus b. Maternal temperature of 99 F c. High uterine fundus displaced to the right d. Intense vaginal pain unrelieved by analgesics e. Half of a lochia pad saturated in the first hour after birth

a. Soft boggy uterus c. High uterine fundus displaced to the right d. Intense vaginal pain unrelieved by analgesics Assessment findings that may indicate a potential complication in the fourth stage include a soft boggy uterus, high uterine fundus displaced to the right, and intense vaginal pain unrelieved by analgesics. The maternal temperature may be slightly elevated after birth because of the inflammation to tissues, and half of a lochia pad saturated in the first hour after birth is within expected amounts.

A laboring client asks the nurse how she will know that the contraction is at its peak. The nurse explains that the contraction peaks during which stage of measurement? a. The acme b. The interval c. The increment d. The decrement

a. The acme The acme is the peak or period of greatest strength during the middle of a contraction cycle. The interval is the period between the end of the contraction and the beginning of the next. The increment is the beginning of the contraction until it reaches the peak. The decrement occurs after the peak until the contraction ends.

11. In which situation would a baseline fetal heart rate of 160 to 170 bpm be considered a normal finding? a. The fetus is at 30 weeks of gestation. b. The mother has a history of fast labors. c. The mother has been given an epidural block. d. The mother has mild preeclampsia but is not in labor.

a. The fetus is at 30 weeks of gestation. The normal preterm fetus may have a baseline rate slightly higher than the term fetus because of an immature parasympathetic nervous system that does not yet exert a slowing effect on the fetal heart rate (FHR). Fast labors should not alter the FHR normally. Any change in the FHR with an epidural is not considered an expected outcome. Preeclampsia should not cause a normal elevation of the FHR.

Which maternal condition should be considered a contraindication for the application of internal monitoring devices? a. Unruptured membranes b. Cervix dilated to 4 cm c. Fetus has known heart defect d. External monitors currently being used

a. Unruptured membranes To apply internal monitoring devices, the membranes must be ruptured. Cervical dilation of 4 cm would permit the insertion of fetal scalp electrodes and an intrauterine catheter. A compromised fetus should be monitored with the most accurate monitoring devices. The external monitor can be discontinued after the internal ones are applied.

Which can be determined only by electronic fetal monitoring? a. Variability b. Tachycardia c. Bradycardia d. Fetal response to contractions

a. Variability Beat-to-beat variability cannot be determined by auscultation because auscultation provides only an average fetal heart rate (FHR) as it fluctuates. Tachycardia can be determined by any of the FHR monitoring techniques. Bradycardia can be determined by any of the FHR monitoring techniques. The fetal response to the contractions is usually noted by an increase or decrease in fetal heart rate. These can be determined by any of the FHR monitoring techniques.

When using the second Leopolds maneuver in fetal assessment, the nurse would palpate (the): a. both sides of the maternal abdomen. b. lower abdomen above the symphysis pubis. c. both upper quadrants of the maternal abdomen . d. lower abdomen for flexion of the presenting part.

a. both sides of the maternal abdomen. The second Leopolds maneuver involves determining the location of the fetal back and is performed by palpating both sides of the maternal abdomen. Palpating the lower abdomen above the symphysis pubis is the third maneuver. Palpating the upper quadrants of the maternal abdomen is the first maneuver. Palpating the lower abdomen for flexion of the presenting part is the fourth maneuver.

To determine if the client is in true labor, the nurse would assess for changes in: a. cervical dilation. b. amount of bloody show. c. fetal position and station. d. pattern of uterine contractions.

a. cervical dilation. Cervical changes are the only indication of true labor and are used to determine true and false labor. Changes in the amount of bloody show, fetal position and station, and pattern of uterine contractions are unreliable indicators of true labor.

Increasing the infusion rate of nonadditive intravenous fluids can increase fetal oxygenation primarily by: a. expanding the maternal blood volume. b. maintaining a normal maternal temperature. c. preventing normal maternal hypoglycemia. d. increasing the oxygen-carrying capacity of the maternal blood.

a. expanding the maternal blood volume. Filling the mothers vascular system makes more blood available to perfuse the placenta and may correct hypotension. Increasing fluid volume may alter the maternal temperature only if she is dehydrated. Most IV fluids for laboring women are isotonic and do not add extra glucose. Oxygen-carrying capacity is increased by adding more red blood cells.

During labor a vaginal examination should be performed only when necessary because of the risk of: a. infection. b. fetal injury. c. discomfort. d. perineal trauma.

a. infection. Vaginal examinations increase the risk of infection by carrying vaginal microorganisms upward toward the uterus. Properly performed vaginal examinations should not cause fetal injury. Vaginal examinations may be uncomfortable for some women in labor, but that is not the main reason for limiting them. A properly performed vaginal examination should not cause perineal trauma.

An increase in urinary frequency and leg cramps after the 36th week of pregnancy most likely indicates: a. lightening. b. breech presentation. c. urinary tract infection. d. onset of Braxton-Hicks contractions.

a. lightening. As the fetus descends toward the pelvic inlet near the end of pregnancy, increased pelvic pressure occurs, resulting in greater urinary frequency and more leg cramps. Breech presentation does not cause urinary frequency and leg cramps. A urinary tract infection may cause urinary frequency but with burning and would not cause leg cramps. Braxton-Hicks contractions are irregular and mild and occur throughout the pregnancy.

If a woman's fundus is soft 30 minutes after birth, the nurses first response should be to: a. massage the fundus. b. take the blood pressure. c. notify the physician or nurse-midwife. d. place the woman in Trendelenburg position.

a. massage the fundus. The nurses first response should be to massage the fundus to stimulate contraction of the uterus to compress open blood vessels at the placental site, limiting blood loss. The blood pressure is an important assessment to determine the extent of blood loss but is not the top priority. Notification should occur after all nursing measures have been attempted with no favorable results. The Trendelenburg position is contraindicated for this woman at this point. This position would not allow for appropriate vaginal drainage of lochia. The lochia remaining in the uterus would clot and produce further bleeding.

The nurse thoroughly dries the infant immediately after birth primarily to: a. reduce heat loss from evaporation. b. stimulate crying and lung expansion. c. increase blood supply to the hands and feet. d. remove maternal blood from the skin surface.

a. reduce heat loss from evaporation. Infants are wet with amniotic fluid and blood at birth, which accelerates evaporative heat loss. Rubbing the infant does stimulate crying but is not the main reason for drying the infant. The main purpose of drying the infant is to prevent heat loss. Drying the infant after birth does not remove all of the maternal blood.

The primary difference between the labor of a nullipara and that of a multipara is: a. total duration of labor. b. level of pain experienced. c. amount of cervical dilation. d. sequence of labor mechanisms.

a. total duration of labor. Multiparas usually labor more quickly than nulliparas, making the total duration of their labor shorter. The level of pain is individual to the woman, not the number of labors she has experienced. Cervical dilation is the same for all labors. The sequence of labor mechanisms is the same with all labors.

At 5 minutes after birth, the nurse assesses that the neonates heart rate is 96 bpm, respirations are spontaneous, with a strong cry, body posture is flexed with vigorous movement, reflexes are brisk, and there is cyanosis of the hands and feet. What Apgar score will the nurse assign? a. 7 b. 8 c. 9 d. 10

b. 8 The neonate is assigned a score of 1 for heart rate and color and a score of 2 for respiratory effort, muscle tone, and reflex response, for a combined total of 8.

A client whose cervix is dilated to 5 cm is considered to be in which phase of labor? a. Latent phase b. Active phase c. Second stage d. Third stage

b. Active phase The active phase of labor is characterized by cervical dilation of 4 to 7 cm. The latent phase is from the beginning of true labor until 3 cm of cervical dilation. The second stage of labor begins when the cervix is completely dilated until the birth of the baby. The third stage of labor is from the birth of the baby until the expulsion of the placenta.

The health care provider for a laboring patient makes the following entry into the patients record: 3/50%/1. What instruction will the nurse implement with the patient? a. You will need to remain in bed attached to the electronic fetal monitor. b. Breathe with me slowly, in through your nose and out through your mouth. c. I will begin the administration of 1000 mL of IV fluid so you can have an epidural. d. Your partner will need to change into scrub attire to attend the imminent birth.

b. Breathe with me slowly, in through your nose and out through your mouth. This client is in the latent phase of the first stage of labor. Use slow, deep chest breathing patterns early in labor to conserve energy for the upcoming process. There is no mention in the stem that the membranes are ruptured, which may prohibit the patient from ambulating. Ambulating during early labor uses gravity to facilitate fetal descent. This is desired because the head is at 1 station. Epidural placement during early labor may slow down the labor process. There is no indication that birth is imminent because the patient is 3 cm dilated.

Which event is the best indicator of true labor? a. Bloody show b. Cervical dilation and effacement c. Fetal descent into the pelvic inlet d. Uterine contractions every 7 minutes

b. Cervical dilation and effacement The conclusive distinction between true and false labor is that contractions of true labor cause progressive change in the cervix. Bloody show can occur before true labor. Fetal descent can occur before true labor. False labor may have contractions that occur this frequently but is usually inconsistent.

The nurse assesses the amniotic fluid. Which characteristic presents the lowest risk of fetal complications? a. Bloody b. Clear with bits of vernix caseosa c. Green and thick d. Yellow and cloudy with foul odor

b. Clear with bits of vernix caseosa Amniotic fluid should be clear and may include bits of vernix caseosa, the creamy white fetal skin lubricant. Green fluid indicates that the fetus passed meconium before birth. The newborn may need extra respiratory suctioning at birth if the fluid is heavily stained with meconium. Cloudy, yellowish, strong-smelling, or foul-smelling fluid suggests infection. Bloody fluid may indicate partial placental separation.

Which of the following behaviors would be applicable to a nursing diagnosis of risk for injury in a client who is in labor? a. Length of second-stage labor is 2 hours. b. Client has received an epidural for pain control during the labor process. c. Client is using breathing techniques during contractions to maximize pain relief. d. Client is receiving parenteral fluids during the course of labor to maintain hydration.

b. Client has received an epidural for pain control during the labor process. A client who has received medication during labor is at risk for injury as a result of altered sensorium, so this presentation is applicable to the diagnosis. A length of 2 hours for the second stage of labor is within the range of normal. Breathing techniques help maintain control over the labor process. Fluids administered during the labor process are used to prevent potential fluid volume deficit.

Which is the most appropriate method of intrapartum fetal monitoring when a woman has a history of hypertension during pregnancy? a. Continuous auscultation with a fetoscope b. Continuous electronic fetal monitoring c. Intermittent assessment with a Doppler transducer d. Intermittent electronic fetal monitoring for 15 minutes each hour

b. Continuous electronic fetal monitoring Maternal hypertension may reduce placental blood flow through vasospasm of the spiral arteries. Reduced placental perfusion is best assessed with continuous electronic fetal monitoring to identify patterns associated with this condition. It is not practical to provide continuous auscultation with a fetoscope. This fetus needs continuous monitoring because it is at high risk for complications.

Which interventions should be performed in the birth room to facilitate thermoregulation of the newborn? (Select all that apply.) a. Place the infant covered with blankets in the radiant warmer. b. Dry the infant off with sterile towels. c. Place stockinette cap on infants head. d. Bathe the newborn within 30 minutes of birth. e. Remove wet linen as needed.

b. Dry the infant off with sterile towels. c. Place stockinette cap on infants head. e. Remove wet linen as needed. Following birth, the newborn is at risk for hypothermia. Therefore, nursing interventions are aimed at maintaining warmth. Drying the infant off, in addition to maintaining warmth, helps stimulate crying and lung expansion, which helps in the transition period following birth. Placing a cap on the infants head helps prevent heat loss. Removal of wet linens helps minimize further heat loss caused by exposure. Newborns should not be covered while in a radiant warmer with blankets because this will impede birth of heat transfer. Bathing a newborn should be delayed for at least a few hours so that the newborn temperature can stabilize during the transition period.

Which mechanism of labor occurs when the largest diameter of the fetal presenting part passes the pelvic inlet? a. Extension b. Engagement c. Internal rotation d. External rotation

b. Engagement Engagement occurs when the presenting part fully enters the pelvic inlet. Extension occurs when the fetal head meets resistance from the tissues of the pelvic floor and the fetal neck stops under the symphysis. This causes the fetal head to extend. Internal rotation occurs when the fetus enters the pelvic inlet. The rotation allows the longest fetal head diameter to conform to the longest diameter of the maternal pelvis. External rotation occurs after the birth of the head. The head then turns to the side so the shoulders can internally rotate and are positioned with their transverse diameter in the anteroposterior diameter of the pelvic outlet.

The nurse is assessing a client in the active phase of labor. What should the nurse expect during this phase? a. The client is sociable and excited. b. The client is requesting pain medication. c. The client begins to experience the urge to push. d. The client experiences loss of control and irritability.

b. The client is requesting pain medication. During the active phase of labor, contraction intensity and discomfort increase to the point where women often request pain medication. Sociability and excitability occur during the latent phase. The urge to push occurs at the end of the transition phase or the second stage of labor. Loss of control and irritability occur during the transition phase of labor.

A gravida 1, para 0, 38 weeks gestation is in the transition phase of labor with SROM and is very anxious. Vaginal exam, 8 cm, 100% effaced, 1 station vertex presentation. She wants the nurse to keep checking her by performing repeated vaginal exams because she is sure that she is progressing rapidly. What is the best response that the nurse can provide to this client at this time? a. Performing more frequent vaginal exams will not make the labor go any quicker. b. Even though she is in transition, frequent vaginal exams must be limited because of the potential for infection. c. Tell the client that she will check every 30 minutes. d. Medicate the client as needed for anxiety so that the labor can progress.

b. Even though she is in transition, frequent vaginal exams must be limited because of the potential for infection. Data reveals a primipara in labor who is in transition (8 to 10 cm) with ruptured membranes. At this point, vaginal exams should be limited until the client feels further pressure and/or has increased bloody show, indicating fetal descent. Telling the client that performing more frequent vaginal exams will not make the labor go any quicker would not be therapeutic because this does not address clients anxiety. Telling the client that the nurse will continue checking every 30 minutes without adequate clinical indication is not the standard of care. Medicating the client is not an appropriate intervention at this time because effective communication will help alleviate stress, and the use of medications during transition may affect maternal and/or fetal well-being during birth.

The nurse is caring for a low-risk client in the active phase of labor. At which interval should the nurse assess the fetal heart rate? a. Every 15 minutes b. Every 30 minutes c. Every 45 minutes d. Every 1 hour

b. Every 30 minutes For the fetus at low risk for complications, guidelines for frequency of assessments are at least every 30 minutes during the active phase of labor. 15-minute assessments would be appropriate for a fetus at high risk. 45-minute assessments during the active phase of labor are not frequent enough to monitor for complications. 1-hour assessments during the active phase of labor are not frequent enough to monitor for complications.

A laboring patient states to the nurse, I have to push! What is the next nursing action? a. Contact the health care provider. b. Examine the patients cervix for dilation. c. Review with her how to bear down with contractions. d. Ask her partner to support her head with each push.

b. Examine the patients cervix for dilation. When the cervix is completely dilated, the head can descend through the pelvis and stimulate the Ferguson, or pushing, reflex. Cervical dilation must first be confirmed because premature pushing efforts may result in cervical edema and corresponding delay in dilation. Once complete dilation has been confirmed, the nurse can notify the health care provider. Teaching positioning and pushing efforts is accomplished once complete dilation has been confirmed.

Which factor ensures that the smallest anterior-posterior diameter of the fetal head enters the pelvis? a. Station b. Flexion c. Descent d. Engagement

b. Flexion The anterior-posterior diameter of the head varies with how much it is flexed. In the most favorable situation, the head is fully flexed and the anterior-posterior diameter is the suboccipitobregmatic, averaging 9.5 cm. The station is the relationship of the fetal presenting part to the level of the ischial spine. Descent is the moving of the fetus through the birth canal. Engagement occurs when the largest diameter of the fetal presenting part has passed the pelvic outlet.

When taking care of a client in labor who is not considered to be at risk, which assessments should be included in the plan of care? (Select all that apply.) a. Check the DTR each shift. b. Monitor and record vital signs frequently during the course of labor. c. Document the FHR pattern, noting baseline and response to contraction patterns. d. Indicate on the EFM tracing when maternal position changes are done. e. Provide food, as tolerated, during the course of labor.

b. Monitor and record vital signs frequently during the course of labor. c. Document the FHR pattern, noting baseline and response to contraction patterns. d. Indicate on the EFM tracing when maternal position changes are done. Nursing care of the normal laboring client would include monitoring and documentation of vital signs as part of the labor assessment, documentation the FHR, checking patterns to look for assurance of fetal well-being by evaluating baseline and the fetal response to contraction patterns, and noting any position changes on the monitor tracing to evaluate the fetal response. Providing dietary offerings during the course of labor is not part of the nursing care plan because the introduction of food may lead to nausea and vomiting in response to the labor process and might affect the mode of birth.

The nurse is teaching a group of nursing students about factors that have a role in starting labor. Which should the nurse include in the teaching session? (Select all that apply.) a. Progesterone levels become higher than estrogen levels. b. Natural oxytocin in conjunction with other substances plays a role. c. Stretching, pressure, and irritation of the uterus and cervix increase. d. The secretion of prostaglandins from the fetal membranes decreases.

b. Natural oxytocin in conjunction with other substances plays a role. c. Stretching, pressure, and irritation of the uterus and cervix increase. Factors that appear to have a role in starting labor include the following: (1) natural oxytocin plays a part in labors initiation in conjunction with other substances; and (2) stretching, pressure, and irritation of the uterus and cervix increase as the fetus reaches term size. The progesterone levels drop and estrogen levels increase. There is an increase in the secretion of prostaglandins from the fetal membranes.

On admission to the labor and birth unit, a 38-year-old female, gravida 4, para 3, at term in early labor is found to have a transverse lie on vaginal examination. What is the priority intervention at this time? a. Perform a vaginal exam to denote progress. b. Notify the health care provider. c. Initiate parenteral therapy. d. Apply oxygen via nasal cannula at 8 L/min.

b. Notify the health care provider. A transverse lie is considered to be an abnormal presentation so the physician should be notified and the process of a C section as the birth method should be initiated. The information provided relative to transverse lie was found on vaginal exam. At this point, the priority is to prepare for a surgical birth because assessment data also indicate that the client is in early labor; thus, a vaginal birth is not imminent. Although initiating parenteral therapy will be required, it is not the priority at this time. Application of oxygen is not required because there is no evidence of fetal or maternal distress.

28. The examiner indicates to the labor nurse that the fetus is in the left occiput anterior (LOA) position. To facilitate the labor process, how will the nurse position the laboring patient? a. On her back b. On her left side c. On her right side d. On her hands and knees

b. On her left side LOA is the desired fetal position for the birthing process. Positioning the patient on her left side will accomplish two objectives: (1) by the use of gravity, the fetus will most likely stay in the LOA position; and (2) increase perfusion of the placenta and increase oxygen to the fetus. Positioning the patient on her back decreases placental perfusion. Positioning on her right may facilitate internal rotation and move the fetus out of the LOA position. The hands and knees position is reserved to decrease cord compression, facilitate the fetus out of a posterior position, or increase oxygenation in the presence of hypoxia. Because none of these conditions are present, there is no need to implement this position.

The nurse notes a concerning fetal heart rate pattern for a patient in active labor. The health care provider has prescribed the placement of a Foley catheter. What priority nursing action will the nurse implement when placing the catheter? a. Place the catheter as quickly as possible. b. Place a small pillow under the patients left hip. c. Omit the use of a cleansing agent, such as Betadine. d. Set up the catheter tray before positioning the patient.

b. Place a small pillow under the patients left hip. To promote placental function, the nurse can place a small pillow or rolled blanket under the patients left hip to shift the weight of the uterus off the aorta and inferior vena cava. Catheter placement is a sterile procedure, with very prescribed steps. Placing the catheter quickly might lead to skipping a step and place the patient at risk for infection. Use of a cleansing agent, such as Hibiclens or Betadine, is included in the catheter placement procedure to ensure a sterile area for placement. Setting up the catheter tray before positioning the patient is the standard of care.

The labor nurse is reviewing the cardinal maneuvers with a group of nursing students. Which maneuver will immediately follow the birth of the baby's head? a. Expulsion b. Restitution c. Internal rotation d. External rotation

b. Restitution After the head emerges, it realigns with the shoulders (restitution). External rotation occurs as the fetal shoulders rotate internally, aligning their transverse diameter with the anteroposterior diameter of the pelvic outlet. Expulsion occurs when the baby is completely delivered. Internal rotation occurs prior to birth of the head.

Which nursing action is correct when initiating electronic fetal monitoring? a. Lubricate the tocotransducer with an ultrasound gel. b. Securely apply the tocotransducer with a strap or belt. c. Inform the client that she should remain in the semi-Fowler position. d. Determine the position of the fetus before attaching the electrode to the maternal abdomen.

b. Securely apply the tocotransducer with a strap or belt. The tocotransducer should fit snugly on the abdomen to monitor uterine activity accurately. The tocotransducer does not need gel to operate appropriately. The client should be encouraged to move around during labor. The tocotransducer should be placed at the fundal area of the uterus.

Which statement correctly describes the nurses responsibility related to electronic monitoring? a. Report abnormal findings to the physician before initiating corrective actions. b. Teach the woman and her support person about the monitoring equipment and discuss any of their questions. c. Document the frequency, duration, and intensity of contractions measured by the external device. d. Inform the support person that the nurse will be responsible for all comfort measures when the electronic equipment is in place.

b. Teach the woman and her support person about the monitoring equipment and discuss any of their questions. Teaching is an essential part of the nurses role. Corrective actions should be initiated first to correct abnormal findings as quickly as possible. Electronic monitoring will record the contractions and FHR response. The support person should still be encouraged to assist with the comfort measures.

A client is receiving oxytocin (Pitocin) to induce labor. The uterine contractions have become persistently hypertonic and the infusion is stopped. The health care provider has prescribed a tocolytic to stop contractions. Which medication should the nurse be prepared to administer? a. Naloxone (Narcan) b. Terbutaline (Brethine) c. Ephedrine d. Diphenhydramine (Benadryl)

b. Terbutaline (Brethine) A tocolytic drug, such as terbutaline (0.125 to 0.25 mg IV or 0.25 mg subcutaneously), may be given to reduce uterine activity. Narcan is a narcotic antagonist. Benadryl is an antihistamine. Ephedrine is a vasopressor used to increase blood pressure.

The primipara at 39 weeks gestation states to the nurse, I can breathe easier now. What is the nurses best response? a. You labor will start any day now since the baby has dropped. b. That process is called lightening. Do you have to urinate more frequently? c. Contact your health care provider when your contractions are every 5 minutes for 1 hour. d. You will likely not feel you baby's movements as much now, so do not be concerned.

b. That process is called lightening. Do you have to urinate more frequently? As the fetus descends toward the pelvic inlet (dropping), the woman notices that she breathes more easily because upward pressure on her diaphragm is reduced. However, increased pressure on her bladder causes her to urinate more frequently. Pressure of the fetal head in the pelvis also may cause leg cramps and edema. Lightening (descent of the fetus toward the pelvic inlet before labor) is most noticeable in primiparas and occurs about 2 to 3 weeks before the natural onset of labor. Instructions for labor, although correct, do not address the patients statement of being able to breathe easier. Fetal movement continues throughout the final weeks of gestation. A decrease in fetal movement is a concerning sign and the health care provider must be notified.

Which nursing assessment indicates that a woman who is in the second stage of labor is almost ready to give birth? a. Bloody mucous discharge increases. b. The vulva bulges and encircles the fetal head. c. The membranes rupture during a contraction. d. The fetal head is felt at 0 station during the vaginal examination.

b. The vulva bulges and encircles the fetal head. A bulging vulva that encircles the fetal head describes crowning, which occurs shortly before birth. Bloody show occurs throughout the labor process and is not an indication of an imminent birth. Rupture of membranes can occur at any time during the labor process and does not indicate an imminent birth. Birth of the head occurs when the station is +4. A 0 station indicates engagement.

When the mothers membranes rupture during active labor, the fetal heart rate should be observed for the occurrence of which periodic pattern? a. Early decelerations b. Variable decelerations c. Nonperiodic accelerations d. Increase in baseline variability

b. Variable decelerations When the membranes rupture, amniotic fluid may carry the umbilical cord to a position where it will be compressed between the maternal pelvis and the fetal presenting part, resulting in a variable deceleration pattern. Early declarations are considered reassuring; they are not a concern after rupture of membranes. Accelerations are considered reassuring; they are not a concern after rupture of membranes. Increase in baseline variability is not an expected occurrence after the rupture of membranes.

The husband of a laboring woman asks the nurse how he can help his wife throughout the first stage of labor. The nurse informs him that in addition to all that hes doing now, he could tell her when the contractions are: a. 2 minutes apart. b. at their acme. c. at their increment. d. at their decrement.

b. at their acme. When the contraction is most intense, the coach can tell the laboring woman that this contraction will be over soon to help her remain focused. Describing the frequency of the contractions is not usually helpful. The increment occurs as the contraction begins in the fundus and spreads through the uterus. Calling attention to this phase may cause the woman to become tense. The woman does not need anyone to tell her that the contraction is decreasing in intensity.

A woman who is gravida 3, para 2, enters the intrapartum unit. The most important nursing assessments are: a. contraction pattern, amount of discomfort, and pregnancy history. b. fetal heart rate, maternal vital signs, and the woman's nearness to birth. c. last food intake, when labor began, and cultural practices the couple desires. d. identification of ruptured membranes, the woman's gravida and para, and her support person.

b. fetal heart rate, maternal vital signs, and the woman's nearness to birth. All options describe relevant intrapartum nursing assessments, but the focus assessment has priority. If the maternal and fetal conditions are normal and birth is not imminent, other assessments can be performed in an unhurried manner. Contraction pattern, amount of discomfort, and pregnancy history are important nursing assessments but do not take priority if the birth is imminent. Last food intake, when labor began, and cultural practices the couple desires is an assessment that can occur later in the admission process, if time permits. Identification of ruptured membranes, the woman's gravida and para, and her support person are assessments that can occur later in the admission process if time permits.

A nursing priority during admission of a laboring client who has not had prenatal care is: a. obtaining admission labs. b. identifying labor risk factors. c. discussing her birth plan choices. d. explaining importance of prenatal care.

b. identifying labor risk factors. When a client has not had prenatal care, the nurse must determine through interviewing and examination the presence of any pregnancy or labor risk factors, obtain admission labs, and discuss birth plan choices. Explaining the importance of prenatal care can be accomplished after the patients history has been completed.

At 1 minute after birth, the nurse assesses the newborn to assign an Apgar score. The apical heart rate is 110 bpm, and the infant is crying vigorously with the limbs flexed. The infants trunk is pink, but the hands and feet are blue. The Apgar score for this infant is: a. 7. b. 8. c. 9. d. 10.

c. 9. The Apgar score is 9 because 1 point is deducted from the total score of 10 for the infants blue hands and feet. The baby received 2 points for each of the categories except color. Because the infants hands and feet were blue, this category is given a grade of 1. The baby received 2 points for each of the categories except color. Because the infants hands and feet were blue, this category is given a grade of 1. The infant had 1 point deducted because of the blue color of the hands and feet.

The nurse is assessing the duration of a clients labor contractions. Which action does the nurse implement to assess the duration of labor contractions? a. Assess the strongest intensity of each contraction. b. Assess uterine relaxation between two contractions. c. Assess from the beginning to the end of each contraction. d. Assess from the beginning of one contraction to the beginning of the next.

c. Assess from the beginning to the end of each contraction. Duration of labor contractions is the average length of contractions from beginning to end. Assessing the strongest intensity of each contraction assesses the strength or intensity of the contractions. Assessing uterine relaxation between two contractions is the interval of the contraction phase. Assessing from the beginning of one contraction to the beginning of the next is the frequency of the contractions.

The health care provider has asked the nurse to prepare for an amniotomy. What is the nurses priority action with this procedure? a. Perform Leopolds maneuvers. b. Determine the color of the amniotic fluid. c. Assess the fetal heart rate immediately after the procedure. d. Prepare the patient for a change in her pain level after the procedure.

c. Assess the fetal heart rate immediately after the procedure. An amniotomy is the artificial rupture of the membranes performed with an AmniHook inserted through the cervix. The FHR is assessed for at least 1 minute when the membranes rupture. The umbilical cord could be displaced in a large fluid gush, resulting in compression and interruption of blood flow through the cord. Leopold's maneuvers should be performed before the amniotomy, which will give an indication of fetal position and station. Color of the fluid can indicate fetal status; however, circulatory assessment is the priority. If the patient is in active labor, a decrease in the amount of amniotic fluid will result in increased intensity of contractions. There is no information in the stem to indicate that the patient is in labor.

A woman arrives to the labor and birth unit at term. She is greeted by a staff nurse and a nursing student. The student reviews the initial intake assessment with the staff nurse. Which action will the staff nurse have to correct? a. Obtain a fetal heart rate. b. Determine the estimated due date. c. Auscultate anterior and posterior breath sounds. d. Ask the client when she last had something to eat.

c. Auscultate anterior and posterior breath sounds. On admission to the labor and birth unit, a focused assessment is performed. This includes the following: names of mother and support person(s); name of her physician or nurse-midwife if she had prenatal care; number of pregnancies and prior births, including whether the birth was vaginal or cesarean; status of membranes; expected date of birth; problems during this or other pregnancies; allergies to medications, foods, or other substances; time and type of last oral intake; maternal vital signs and FHR; and painlocation, intensity, factors that intensify or relieve, duration, whether constant or intermittent, and whether the pain is acceptable to the woman. Generally, women of childbearing years are healthy and auscultation of lung sounds can be delayed until the initial intake assessment has been completed.

When a nonreassuring pattern of the fetal heart rate is noted and the client is lying on her left side, which nursing action is indicated? a. Lower the head of the bed. b. Place a wedge under the left hip. c. Change her position to the right side. d. Place the mother in Trendelenburg position.

c. Change her position to the right side. Repositioning on the opposite side may relieve compression on the umbilical cord and improve blood flow to the placenta. Lowering the head of the bed would not be the first position change choice. The woman is already on her left side, so a wedge on that side would not be an appropriate choice. Repositioning to the opposite side is the first intervention. If unsuccessful with improving the FHR pattern, further changes in position can be attempted; the Trendelenburg position might be the choice.

Which assessment finding would cause a concern for a client who had delivered vaginally? a. Estimated blood loss (EBL) of 500 mL during the birth process b. White blood cell count of 28,000 mm3 postbirth c. Client complains of fingers tingling d. Client complains of thirst

c. Client complains of fingers tingling A clients complaint of fingers tingling may represent respiratory alkalosis due to hyperventilation breathing patterns during labor. As such it requires intervention by the nurse to have the client slow breathing down and restore normal carbon dioxide levels.

The fetal heart rate baseline increases 20 bpm after vibroacoustic stimulation. The best interpretation of this is that the fetus is showing: a. a worsening hypoxia. b. progressive acidosis. c. a reassuring response. d. parasympathetic stimulation.

c. a reassuring response The fetus with adequate reserve for the stress of labor will usually respond to vibroacoustic stimulation with a temporary increase in the fetal heart rate (FHR) baseline. An increase in the FHR with stimulation does not indicate hypoxia. An increase in the FHR after stimulation is reassuring and does not indicate acidosis. An increase in the FHR after stimulation is a reassuring pattern and does not indicate problems with the parasympathetic nervous system.

The nurse is explaining to a group of nursing students what occurs during active labor as the uterus contracts. Which statement explains the maternal-fetal exchange of oxygen and waste products during a contraction? a. Is not significantly affected b. Increases as blood pressure decreases c. Diminishes as the spiral arteries are compressed d. Continues except when placental functions are reduced

c. Diminishes as the spiral arteries are compressed During labor contractions, the maternal blood supply to the placenta gradually stops as the spiral arteries supplying the intervillous space are compressed by the contracting uterine muscle. The exchange of oxygen and waste products is affected by contractions. The exchange of oxygen and waste products decreases. The maternal blood supply to the placenta gradually stops with contractions.

The nurse notes that a client who has given birth 1 hour ago is touching her infant with her fingertips and talking to him softly in high-pitched tones. Based on this observation, which action should the nurse take? a. Request a social service consult for psychosocial support. b. Observe for other signs that the mother may not be accepting of the infant. c. Document this evidence of normal early maternal-infant attachment behavior. d. Determine whether the mother is too fatigued to interact normally with her infant.

c. Document this evidence of normal early maternal-infant attachment behavior. Normal early maternal-infant behaviors are tentative and include fingertip touch, eye contact, and using a high-pitched voice when talking to the infant. There is no indication at this point that a social service consult is necessary. The signs are of normal attachment behavior. These are signs of normal attachment behavior; no other assessment is necessary at this point. The mother may be fatigued but is interacting with the infant in an expected manner.

When the deceleration pattern of the fetal heart rate mirrors the uterine contraction, which nursing action is indicated? a. Reposition the client. b. Apply a fetal scalp electrode. c. Record this reassuring pattern. d. Administer oxygen by nasal cannula.

c. Record this reassuring pattern. The periodic pattern described is early deceleration that is not associated with fetal compromise and requires no intervention. It is a reassuring pattern. Repositioning the client, applying a fetal scalp electrode, or administering oxygen would be interventions done for nonreassuring patterns.

If the position of a fetus in a cephalic presentation is right occiput anterior, the nurse should assess the fetal heart rate in which quadrant of the maternal abdomen? a. Right upper b. Left upper c. Right lower d. Left lower

c. Right lower If the fetus is in a right occiput anterior position, the fetal spine will be on the mothers right side. The best location to hear the fetal heart rate is through the fetal shoulder, which would be in the right lower quadrant. The right upper, left upper, and left lower areas are not the best locations for assessing the fetal heart rate in this case.

The nurse assess a laboring patients contraction pattern and notes the frequency at every 3 to 4 minutes, duration 50 to 60 sections, and the intensity is moderate by palpation. What is the most accurate documentation for this contraction pattern? a. Stage 1, latent phase b. Stage 2, latent phase c. Stage 1, active phase d. Stage 2, active phase

c. Stage 1, active phase In the active phase of stage 1, contractions are about 2 to 5 minutes apart, with a duration of about 40 to 60 seconds and an intensity that ranges from moderate to strong. During the latent phase of stage 1, the interval between contractions shortens until contractions are about 5 minutes apart. Duration increases to 30 to 40 seconds by the end of the latent phase. During stage 2, latent phase, the woman is resting and preparing to push; she likely has not experienced the Ferguson reflex. She is actively bearing down during the active phase of the second stage.

A client at 40 weeks gestation should be instructed to go to a hospital or birth center for evaluation when she experiences: a. fetal movement. b. irregular contractions for 1 hour. c. a trickle of fluid from the vagina. d. thick pink or dark red vaginal mucus.

c. a trickle of fluid from the vagina. A trickle of fluid from the vagina may indicate rupture of the membranes, requiring evaluation for infection or cord compression. The lack of fetal movement needs further assessment. Irregular contractions are a sign of false labor and do not require further assessment. Bloody show may occur before the onset of true labor. It does not require professional assessment unless the bleeding is pronounced.

The nurse assists the midwife during a vaginal examination of the client in labor. What does the nurse recognize as the primary reason that a vaginal exam is done at this time? a. To apply internal monitoring electrodes b. To assess for Goodell's sign c. To determine cervical dilation and effacement d. To determine strength of contractions

c. To determine cervical dilation and effacement The primary purpose of a vaginal exam during labor is to determine cervical dilation and effacement and fetal descent. Goodell's sign is assessed in early pregnancy, not during labor. Although application of monitoring electrodes is done by entering the vagina, it is not the primary purpose of a vaginal exam. Vaginal exams are not done to determine the strength of contractions.

The nurse is preparing to perform Leopolds maneuvers. Why are Leopolds maneuvers used by practitioners? a. To determine the status of the membranes b. To determine cervical dilation and effacement c. To determine the best location to assess the fetal heart rate d. To determine whether the fetus is in the posterior position

c. To determine the best location to assess the fetal heart rate Leopolds maneuvers are often performed before assessing the fetal heart rate (FHR). These maneuvers help identify the best location to obtain the FHR. A Nitrazine or ferning test can be performed to determine the status of the fetal membranes. Dilation and effacement are best determined by vaginal examination. Assessment of fetal position is more accurate with vaginal examination.

If a notation on the clients health record states that the fetal position is LSP, this means that the: a. head is in the right posterior quadrant of the pelvis. b. head is in the left anterior quadrant of the pelvis. c. buttocks are in the left posterior quadrant of the pelvis. d. buttocks are in the right upper quadrant of the abdomen.

c. buttocks are in the left posterior quadrant of the pelvis. LSP explains the position of the fetus in the maternal pelvis. L = left side of the pelvis, S = sacrum (fetus is in breech presentation), P = posterior quadrants of the pelvis. When the head is in the right posterior quadrant of the pelvis, the position is ROP. When the head is in the left anterior quadrant of the pelvis, the position is ROA. When the buttocks are in the upper quadrant of the abdomen, the position would be ROA, ROP, LOA, LOP, LOT, or ROT.

Proper placement of the tocotransducer for electronic fetal monitoring is: a. inside the uterus. b. on the fetal scalp. c. over the uterine fundus. d. over the mothers lower abdomen.

c. over the uterine fundus. The tocotransducer monitors uterine activity and should be placed over the fundus, where the most intensive uterine contractions occur. The tocotransducer is for external use. The tocotransducer monitors uterine contractions. The most intensive uterine contractions occur at the fundus; this is the best placement area.

A nullipara client has progressed to the active phase of labor. The nurse understands that this phase of labor, on the average, for a nullipara will last how long? a. 50 minutes b. hours c. 6 to 7 hours d. 8 to 10 hours

d. 8 to 10 hours The active phase of labor for a nullipara lasts 8 to 10 hours. The second phase of labor lasts 50 minutes for a nullipara. The transition phase lasts hours for a nullipara. A multiparas active phase of labor is 6 to 7 hours.

Which nursing diagnosis would take priority in the care of a primipara client with no visible support person in attendance who has entered the second stage of labor after a first stage of labor lasting 4 hours? a. Fluid volume deficit (FVD) related to fluid loss during labor and birth process b. Fatigue related to length of labor requiring increased energy expenditure c. Acute pain related to increased intensity of contractions d. Anxiety related to imminent birth process

d. Anxiety related to imminent birth process A primipara is experiencing the birthing event for the first time and may experience anxiety because of fear of the unknown. It would be important to recognize this because the client is alone in the labor-birth room and will need additional support and reassurance. Although FVD may occur as a result of fluid loss, prospective management of labor clients includes the use of parenteral fluid therapy; the client should be monitored for FVD and, if symptoms warrant, receive intervention. Because the client has been in labor for 4 hours, this is not considered to be a prolonged labor pattern for a primipara client. Although the client may be tired, this nursing diagnosis would not be a priority unless there were other symptoms manifested. Because the client is entering the second stage of labor, she will be allowed to push with contractions. Thus, in terms of pain management, medication will not be administered at this time because of imminent birth.

The nurse examines a primiparas cervix at 8-9/100%/+2; it is tight against the fetal head. The patient reports a strong urge to bear down. What is the nurses priority action? a. Palpate her bladder for fullness. b. Assess the frequency and duration of her contractions. c. Determine who will stay with the patient for the birth. d. Encourage the patient to exhale in short breaths with contractions.

d. Encourage the patient to exhale in short breaths with contractions. Teach the woman to exhale in short breaths if pushing is likely to injure her cervix or cause cervical edema. Pushing against a cervix that does not easily yield to pressure from the presenting part may result in cervical edema, which can block labor progress or cause cervical lacerations. A full bladder may impede the progress of labor. Although this is an important nursing action, it does not address the patients urge to push. This patient is in the transition phase of the first stage of labor. Her contractions will be every 2 to 3 minutes and last 60 to 90 seconds. Determining the frequency and duration of the contractions does not add to the known assessment data for this patient. Determining who will attend the birth, although nice to know, does not address her urge to push.

The gynecologist performs an amniotomy. What will the nurses role include immediately following the procedure? a. Assessing for ballottement b. Conducting a pH and/or fern test c. Labeling of specimens for chromosomal analysis d. Recording the character and amount of amniotic fluid

d. Recording the character and amount of amniotic fluid An amniotomy is a procedure in which the amniotic sac is deliberately ruptured. It is important to note and record the character and amount of amniotic fluid following this procedure. Assessing for ballottement is not indicated. Conducting a pH or fern test is not needed because an amniotomy releases amniotic fluid. An amniocentesis, not an amniotomy, is used to collect a specimen for chromosomal analysis.

Which client at term should go to the hospital or birth center the soonest after labor begins? a. Gravida 2, para 1, who lives 10 minutes away b. Gravida 1, para 0, who lives 40 minutes away c. Gravida 2, para 1, whose first labor lasted 16 hours d. Gravida 3, para 2, whose longest previous labor was 4 hours

d. Gravida 3, para 2, whose longest previous labor was 4 hours Multiparous women usually have shorter labors than do nulliparous women. The woman described in option D is multiparous with a history of rapid labors, increasing the likelihood that her infant might be born in uncontrolled circumstances. A gravida 2 would be expected to have a longer labor than the gravida in option C. The fact that she lives close to the hospital allows her to stay home for a longer period of time. A gravida 1 will be expected to have the longest labor. The gravida 2 would be expected to have a longer labor than the gravida 3, especially because her first labor was 16 hours.

The nurse is directing an unlicensed assistive personnel (UAP) to take maternal vital signs between contractions. Which statement is the best rationale for assessing maternal vital signs between contractions? a. Vital signs taken during contractions are not accurate. b. During a contraction, assessing fetal heart rate is the priority. c. Maternal blood flow to the heart is reduced during contractions. d. Maternal circulating blood volume increases temporarily during contractions.

d. Maternal circulating blood volume increases temporarily during contractions. During uterine contractions, blood flow to the placenta temporarily stops, causing a relative increase in the mothers blood volume, which in turn temporarily increases blood pressure and slows the pulse. Vital signs are altered by contractions but are considered accurate for a period of time. It is important to monitor the fetal response to contractions, but the question is concerned with the maternal vital signs. Maternal blood flow is increased during a contraction.

Which should the nurse recognize as being associated with fetal compromise? a. Active fetal movements b. Fetal heart rate in the 140s c. Contractions lasting 90 seconds d. Meconium-stained amniotic fluid

d. Meconium-stained amniotic fluid When fetal oxygen is compromised, relaxation of the rectal sphincter allows passage of meconium into the amniotic fluid. Active fetal movement is an expected occurrence. The expected FHR range is 120 to 160 bpm. The fetus should be able to tolerate contractions lasting 90 seconds if the resting phase is sufficient to allow for a return of adequate blood flow.

The nurse is concerned that a clients uterine activity is too intense and that her obesity is preventing accurate assessment of the actual intrauterine pressure. Based on this information, which action should the nurse take? a. Reposition the tocotransducer. b. Reposition the Doppler transducer. c. Obtain an order from the health care provider for a spiral electrode. d. Obtain an order from the health care provider for an intrauterine pressure catheter.

d. Obtain an order from the health care provider for an intrauterine pressure catheter. An intrauterine pressure catheter can measure actual intrauterine pressure. The tocotransducer measures the uterine pressure externally; this would not be accurate with an obese client, even with repositioning. A Doppler auscultates the FHR. A scalp electrode (or spiral electrode) measures the fetal heart rate (FHR).

In which situation would it be appropriate to obtain a fetal scalp blood sample to establish fetal well-being? a. The fetus has developed tachycardia related to maternal fever. b. The mother has vaginal bleeding, and the baseline fetal heart rate is decreasing. c. The fetal heart tracing on a preterm fetus shows decreased baseline variability. d. The fetal heart tracing shows a persistent pattern of late decelerations, with normal baseline variability.

d. The fetal heart tracing shows a persistent pattern of late decelerations, with normal baseline variability. The tracing is nonreassuring, and additional assessment is needed regarding the acid-base status of the fetus. Fetal scalp blood sampling is contraindicated with vaginal bleeding, maternal fever, and a preterm fetus.

The nurse sees a pattern on the fetal monitor that looks similar to early decelerations, but the deceleration begins near the acme of the contraction and continues well beyond the end of the contraction. Which nursing action indicates the proper evaluation of this situation? a. This pattern reflects variable decelerations. No interventions are necessary at this time. b. Document this reassuring fetal heart rate pattern but decrease the rate of the intravenous (IV) fluid. c. Continue to monitor these early decelerations, which occur as the fetal head is compressed during a contraction. d. This deceleration pattern is associated with uteroplacental insufficiency, so the nurse acts quickly to improve placental blood flow and fetal oxygen supply.

d. This deceleration pattern is associated with uteroplacental insufficiency, so the nurse acts quickly to improve placental blood flow and fetal oxygen supply. A pattern similar to early decelerations, but the deceleration begins near the acme of the contraction and continues well beyond the end of the contraction, describes a late deceleration. Oxygen should be given via a snug face mask. Position the client on her left side to increase placental blood flow. Variable decelerations are caused by cord compression. A vaginal examination should be performed to identify this potential emergency. This is not a reassuring pattern, so the IV rate should be increased to increase the mothers blood volume. These are late decelerations, not early; therefore, interventions are necessary.

Why is continuous electronic fetal monitoring generally used when oxytocin is administered? a. Fetal chemoreceptors are stimulated. b. The mother may become hypotensive. c. Maternal fluid volume deficit may occur. d. Uteroplacental exchange may be compromised.

d. Uteroplacental exchange may be compromised. The uterus may contract more firmly and the resting tone may be increased with oxytocin use. This response reduces the entrance of freshly oxygenated maternal blood into the intervillous spaces, depleting fetal oxygen reserves. Oxytocin affects the uterine muscles. Hypotension is not a common side effect of oxytocin. All laboring women are at risk for fluid volume deficit; oxytocin administration does not increase the risk.

A 25-year-old primigravida client is in the first stage of labor. She and her husband have been holding hands and breathing together through each contraction. Suddenly, the client pushes her husbands hand away and shouts, Don't touch me! This behavior is most likely: a. abnormal labor. b. a sign that she needs analgesia. c. normal and related to hyperventilation. d. common during the transition phase of labor.

d. common during the transition phase of labor. The transition phase of labor is often associated with an abrupt change in behavior, including increased anxiety and irritability. This change of behavior is an expected occurrence during the transition phase. If she is in the transitional phase of labor, analgesia may not be appropriate if the birth is near. Hyperventilation will produce signs of respiratory alkalosis.

A primigravida at 39 weeks of gestation is observed for 2 hours in the intrapartum unit. The fetal heart rate has been normal. Contractions are 5 to 9 minutes apart, 20 to 30 seconds in duration, and of mild intensity. Cervical dilation is 1 to 2 cm and uneffaced (unchanged from admission). Membranes are intact. The nurse should expect the client to be: a. discharged home with a sedative. b. admitted for extended observation. c. admitted and prepared for a cesarean birth. d. discharged home to await the onset of true labor.

d. discharged home to await the onset of true labor. The situation describes a client with normal assessments who is probably in false labor and will probably not deliver rapidly once true labor begins. The client will probably be discharged, but there is no indication that a sedative is needed. These are all indications of false labor; there is no indication that further assessment or observations are indicated. These are all indications of false labor without fetal distress. There is no indication that a cesarean birth is indicated.

The laboring client asks the nurse how the labor contractions work to dilate the cervix. The best response by the nurse is that labor contractions facilitate cervical dilation by: a. promoting blood flow to the cervix. b. contracting the lower uterine segment. c. enlarging the internal size of the uterus. d. pulling the cervix over the fetus and amniotic sac.

d. pulling the cervix over the fetus and amniotic sac. Effective uterine contractions pull the cervix upward at the same time the fetus and amniotic sac are pushed downward. Blood flow decreases to the uterus during a contraction. The contractions are stronger at the fundus. The internal size becomes smaller with the contractions; this helps push the fetus down.

Uncontrolled maternal hyperventilation during labor results in: a. metabolic acidosis. b. metabolic alkalosis. c. respiratory acidosis. d. respiratory alkalosis.

d. respiratory alkalosis. Rapid deep respirations cause the laboring woman to lose carbon dioxide through exhalation, resulting in respiratory alkalosis.


संबंधित स्टडी सेट्स

Chapter 56 PrepU: Dermatological

View Set

Chapter 4. Activity-Based Costing

View Set

MKTG 410 - Consumer Behavior Key Terms Ch. 1

View Set